LSAT and Law School Admissions Forum

Get expert LSAT preparation and law school admissions advice from PowerScore Test Preparation.

User avatar
 Dave Killoran
PowerScore Staff
  • PowerScore Staff
  • Posts: 5853
  • Joined: Mar 25, 2011
|
#27065
Complete Question Explanation
(The complete setup for this game can be found here: lsat/viewtopic.php?t=8627)

The correct answer choice is (B)

As described earlier, when P is reduced, L cannot be reduced. When L is not reduced, M and R must be reduced, and hence answer choice (B) is correct.
 mgoss
  • Posts: 4
  • Joined: Sep 21, 2019
|
#68324
Hi,

Why wouldn't D also be correct? If P is selected, then L must not be. If L is not, then M and R must be (which is the correct answer). But following the logic chain, If R is selected (which it must be), then N must not be (due to rule 2). If N is not, then S must be. Therefore why would not answer choice D be correct as well.

I am confused because the double not arrows are as follows:

P :dblline: L
N :dblline: S
N :dblline: R

Am I interpreting rule 2 incorrectly?
 Jeremy Press
PowerScore Staff
  • PowerScore Staff
  • Posts: 1000
  • Joined: Jun 12, 2017
|
#68360
Hi,

Yes, you're right--you've incorrectly interpreted the rule governing the relationship between N and S. It's true that if N is selected, then S is not selected. But the logic of that rule allows for the possibility that neither N nor S is selected. In other words, just because N is NOT selected, doesn't mean S IS selected. S not being selected is the necessary condition of the rule, and a necessary condition can occur with or without its sufficient condition occurring. Thus S not being selected can occur when N is selected, but (and this is the key) it can also happen when N is not selected.

I hope this helps!

Jeremy
 mgoss
  • Posts: 4
  • Joined: Sep 21, 2019
|
#68457
Thanks, makes sense now.

Get the most out of your LSAT Prep Plus subscription.

Analyze and track your performance with our Testing and Analytics Package.